LSAT and Law School Admissions Forum

Get expert LSAT preparation and law school admissions advice from PowerScore Test Preparation.

 Administrator
PowerScore Staff
  • PowerScore Staff
  • Posts: 8916
  • Joined: Feb 02, 2011
|
#22926
Complete Question Explanation

Assumption. The correct answer choice is (C)

We have two main premises here. First, we are given the fact that some industrialized countries face the prospect of labor shortages. Second, we are told that many experienced workers will be driven out by mandatory retirement laws. From there we jump to the conclusion that eliminating those mandatory retirement laws would avert the labor shortage, despite the fact that we know nothing about the desire of 65 year olds to continue working if they were given the option. Since this is an assumption question, this is the gap we need to fill.

Answer Choice (A): The argument already addresses this issue by focusing on the experience and productivity of older workers. Further, the skills of the workers are not necessary to avert the labor shortage, just the numbers of workers.

Answer Choice (B): While this may be a different argument against mandatory retirement, it does nothing to help the argument of the stimulus.

Answer Choice (C): This is the correct answer choice. Using the assumption negation technique, if there were not a large number of workers that would continue working past 65 if allowed to do so, the conclusion would be weakened. The fact that the workers would continue working serves as the necessary link between the premises and the conclusion.

Answer Choice (D): Again, while this may be an argument against a mandatory retirement age of 65, it does nothing to help the argument of the stimulus.

Answer Choice (E): This would actually serve to weaken the conclusion of the stimulus. If a large proportion of the officially retired workers are already engaged in gainful employment, eliminating the mandatory retirement age is not going to result in the influx of workers that the stimulus anticipates.
 student987
  • Posts: 28
  • Joined: Apr 09, 2018
|
#45281
Hello. I'm confused about why this is not Assumption-CE. Isn't the argument ("eliminating those mandatory retirement laws would avert the labor shortage") saying that elimination of retirement laws would cause these economies' averting of labor shortages?
 Francis O'Rourke
PowerScore Staff
  • PowerScore Staff
  • Posts: 471
  • Joined: Mar 10, 2017
|
#45282
If you are asking yourself if a term expresses causality, it is a bad habit to insert the phrase "cause a __ing of" in the sentence and see if that works. Doing so can change the original meaning too much to be of any help. For example, compare the following: "he passed the ball to me" and "he caused the passing of the ball to me."

I would not look at this example statement as causal in the same way that we look at causal statements on the LSAT. Instead, you should ask yourself if the speaker is specifically alleging that one object or event directly made the other object or event happen.

On some level almost any transitive verb can be described as expressing causality. Only some jump out at you as clearing arguing that one thing made something else occur. Consider the following: "The successful pass played a roll in our win today." In this example, I am not just arguing that the pass occurred, as I was in the example given in the first paragraph above. Instead I am arguing here that the pass caused something; it was a direct factor in us winning the game.

It is unlikely that the speaker intended the statement in #6 to be understood as causal, as the speaker most likely does not believe that ending the discriminatory practices would directly cause the labor shortages to end.

Why? It is already hinted at earlier in the stimulus that the speaker believes that ending the discriminatory practices would allow people to continue working past the age of 65, and that this allowance would cause the pool of available workers to increase. That merely indirect relationship between ending the practices and ending the labor shortages is why we don't want to think of this conclusion as causal.
 esther913
  • Posts: 23
  • Joined: Apr 13, 2019
|
#64318
Hi,
I would like to ask two questions regarding answer choice (C):

1. What is the proper negation of (C)? I thought it would be " a large number of workers in some industrialized countries would not continue working beyond the age of sixty-five..." but according to the Complete Question Explanation, it seems that I should have negated the "large number" part. :oops: What am I missing?

2. Why is (C) a supporter assumption? I could not find any logical gaps just by reading the stimulus and since there weren’t any rogue elements in the conclusion, I thought this question was asking for a defender assumption.
I diagrammed the conclusion as
eliminate mandatory retirement at 65 :arrow: labor shortages averted

I chose (C) as the correct answer choice because it defended/strengthened the necessary condition of the conclusion ("the labor shortages...would be averted") by stating that workers would indeed continue to work beyond the age of 65 if they were allowed to do so.

However, the Complete Question Explanation states that (C) works as the “necessary link between the premises and the conclusion.” Why is (C) a supporter assumption instead of a defender? I don't understand... :-?

Clarification would be greatly appreciated.
Thank you!
 Adam Tyson
PowerScore Staff
  • PowerScore Staff
  • Posts: 5153
  • Joined: Apr 14, 2011
|
#64338
Thanks for the questions, esther913! Your negation of answer C is perfect - it's just a different way of expressing the same idea. Basically, answer C is a condition claim - IF we didn't have mandatory retirement, THEN a bunch of older folks would keep working. The way to negate a conditional statement is to say that the alleged necessary condition isn't actually necessary. While there is more than one way to do that, my approach would probably be something like "those older people might not keep working even if they were allowed to do so." Whether you focus on the "large number" and say that it won't be a large number, or you focus on the "would continue to work" and say they would not necessarily do so, you're getting the same result. Don't worry if your prephrase isn't identical to our suggestion, as long as you have the right concept covered! The negation should effectively say that the answer choice is not true. You did that!

As to the Supporter vs Defender model, that isn't so much about what type of question it is, but about how you go about solving it. If you see a logical gap in the argument, you prephrase something that closes the gap - a supporter. If you see a weakness, you prephrase something that fights off that weakness - a defender. Either approach should get you the same answer! Our admin reacted to this one as presenting a gap between allowing older folks to work and them actually doing so, and that led to a prephrase based on the supporter model. You (and I) saw it as a weakness - what if the old folks don't decide to keep working? - and that gave you a prephrase based on the defender model. All three of us essentially prephrased the same thing, that the older folks would continue to work if allowed to do so! Supporters and Defenders aren't two different types of Assumption questions - they are two different strategies for attacking those questions. Sometimes one approach is a more obvious choice than another, but neither is "right" or "wrong". It's just about how you react to the stimulus and choose to go about the task!

Good work here! Keep that up!
 esther913
  • Posts: 23
  • Joined: Apr 13, 2019
|
#64357
Thank you so much for the kind encouragement!
I am surprised to hear that my negation of C is perfect :0 . It seems that I am on the right track after all!
Also, thank you for explaining Supporters and Defenders in detail. Now I understand that they are just two different ways to attack Assumption questions.
Thank you for the clarification. :)
User avatar
 annabelle.swift
  • Posts: 54
  • Joined: Sep 01, 2021
|
#91505
I had eliminated C because it said "in some industrialized countries."

How do we know that these industrialized countries overlap with the industrialized countries mentioned in the stimulus? How do we know that C's industrialized countries are even countries that face impending labor shortages?
 Adam Tyson
PowerScore Staff
  • PowerScore Staff
  • Posts: 5153
  • Joined: Apr 14, 2011
|
#91628
The author has to assume that this will occur in those countries that are experiencing labor shortages, annabelle, and if it has to happen in even one country then it must happen in "some" countries. "Some" just means "at least one" in logical terms, and the author has to believe it will happen in at least one place. You're right that they ALSO have to believe that the places where that happens will be ones that are currently facing shortages, but while there may be other assumptions underlying the argument, this assumption is still required! The proof of that is in the negation: what if that turns out to be true in ZERO countries? That is, if that doesn't happen anywhere, the argument will completely fall apart.

Don't eliminate an assumption answer because it fails to completely prove the conclusion of the argument; that's the job of a different answer type, a "justify the conclusion." When you are supposed to select something that is necessary, it can be something that is not enough, by itself, to prove the conclusion, so long as it is necessary in order for the conclusion to be valid.

Get the most out of your LSAT Prep Plus subscription.

Analyze and track your performance with our Testing and Analytics Package.